Is this the correct inverse laplace transform

In summary, the conversation discusses the inverse Laplace transform of a given function and how to avoid using complex numbers when solving for the transform. The correct partial fractions expansion is provided, with the suggestion to use the shifting property to simplify the calculations.
  • #1
Ready2GoXtr
75
0

Homework Statement


Well i did all my work but the forum logged me out and when i submitted it I lost it all
because it wanted me to login again >_<
anyways i need to find the inverse laplace transform of
F(s)=[12*s^2 - (28+2*sqrt(3))*s + 26 + 6*sqrt(3)]/[(s-3)(s^2-2s+2)]

Homework Equations





The Attempt at a Solution



F(s)=K1/(s-3) + K2/(s-1-j) + K2*/(s-1+j) Note * is for conjugate
K1= [12(3)^2 - (28+2*sqrt(3))*(3) + 26 + 6*sqrt(3)]/[(3)^2-2(3)+2]=50/5=10
K1=10
K2=[12(1+j)^2 - (28+2*sqrt(3))*(1+j) + 26 + 6*sqrt(3)]/[(1+j+3)*(1+j-1+j)]
=-1.0231-.3603j
K2=-1.0231-.3603j
K2*=-1.0231+.3603j


F(s)=10/(s-3) + (-1.0231-.3603j)/(s-1-j) + (-1.0231+.3603j)/(s-1+j)

note: (-1.0231-.3603j)=1.0846[tex]\angle[/tex]19.4=1.0846*exp(j*19.4)
note: (-1.0231+.3603j)=1.0846[tex]\angle[/tex]-19.4=1.0846*exp(-j*19.4)
doing the inverse laplace transform
I think the second exponent for the last two terms come from translation in frequency operation for laplace transforms but i could be wrong
it says
for f(t): exp(-a*t)*f(t)
for F(s):F(s+a)

f(t)= 10*exp(3*t) + 1.0846*exp(j*19.4)*exp(-(-1-j)t) + 1.0846*exp(-j*19.4)*exp(-(-1+j)t)

if this is correct how do i get rid of the j's here?
 
Physics news on Phys.org
  • #2
When you calculated K2, you accidentally changed (s-3) to (s+3) in the denominator.

You can use e=cos θ + j sin θ to eliminate the j's, but you might find it easier and a lot less tedious to avoid them in the first place. When you do the partial fractions expansion, use the form

[tex]F(s) = \frac{A}{s-3} + \frac{B s + C}{s^2-2s+2}[/tex]

The constants A, B, and C will all be real. To find the inverse Laplace transform of the second term, express it in the form

[tex]\frac{B s + C}{s^2-2s+2} = \frac{B(s-1)}{(s-1)^2+1} + \frac{B+C}{(s-1)^2+1}[/tex]

and use the shifting property you mentioned.
 

Related to Is this the correct inverse laplace transform

1. What is an inverse Laplace transform?

An inverse Laplace transform is a mathematical operation that takes a function in the complex frequency domain and converts it back into a function in the time domain. It is the reverse operation of the Laplace transform, which converts a function from the time domain to the frequency domain.

2. How do you know if you have the correct inverse Laplace transform?

To determine if an inverse Laplace transform is correct, you can use the Laplace transform tables to check if the resulting function matches any known transforms. Additionally, you can check if the transform satisfies the properties of linearity, time shifting, and frequency shifting.

3. Can there be multiple correct inverse Laplace transforms for a single function?

Yes, there can be multiple correct inverse Laplace transforms for a single function. This is because the Laplace transform is not a one-to-one function, meaning multiple functions can have the same transform. However, all valid inverse Laplace transforms should satisfy the properties of linearity, time shifting, and frequency shifting.

4. Are there any common mistakes when finding the inverse Laplace transform?

Some common mistakes when finding the inverse Laplace transform include forgetting to account for the initial conditions, using the wrong Laplace transform table, and incorrectly applying the properties of linearity, time shifting, and frequency shifting. It is important to carefully check your work and make sure all steps are correct.

5. Is there a general formula for finding the inverse Laplace transform?

No, there is not a general formula for finding the inverse Laplace transform. However, there are various techniques such as partial fraction decomposition, convolution, and use of the Laplace transform tables that can be used to find the inverse transform of a function. The specific technique used will depend on the form of the function in the frequency domain.

Similar threads

  • Engineering and Comp Sci Homework Help
Replies
3
Views
1K
  • Engineering and Comp Sci Homework Help
Replies
4
Views
1K
  • Engineering and Comp Sci Homework Help
Replies
3
Views
2K
  • Engineering and Comp Sci Homework Help
Replies
10
Views
2K
  • Engineering and Comp Sci Homework Help
Replies
3
Views
1K
  • Calculus and Beyond Homework Help
Replies
10
Views
2K
  • Engineering and Comp Sci Homework Help
Replies
4
Views
1K
  • Engineering and Comp Sci Homework Help
Replies
5
Views
2K
  • Engineering and Comp Sci Homework Help
Replies
2
Views
1K
  • Differential Equations
Replies
17
Views
1K
Back
Top